0 Daumen
655 Aufrufe

Aufgabe:

\( \int \limits_{Q} \int y \cos \left(x^{2}\right) d(x, y) \) mit \( Q=\left\{(x, y) \in \mathbb{R}^{2}: 0 \leq y \leq 1, y^{2} \leq x \leq 1\right\} \)


Problem/Ansatz:

Ich habe die Reihenfolge der Integrale geändert (Satz von Fubini)(Habe erst nach y und danach nach x integriert) und habe nun nach dem Lösen des Integrals und Einsetzen der letzten Grenze [y^2,1] bekommen :

[1/4 sin(x^2)]->Grenzen [y^2,1] = 1/4*sin(1^2)-1/4*sin(y^4) = 1/4 -1/4*sin(y^4)


Laut der Musterlösung ist dieses Ergebnis falsch.

Avatar von

1 Antwort

+1 Daumen
 
Beste Antwort

Aloha :)

Zu berechnen ist das Integral$$I=\int\limits_0^1dy\int\limits_{y^2}^1dx\,y\cos(x^2)$$Die Untegrenze von \(x\) ist \(y^2\), daher muss während der Integration über \(dx\) der Wert von \(y\) festgehalten werden. Das heißt, du kannst hier die Integrationsreihenfolge nicht ändern, ohne auch die Integrationsgrenzen anzupassen. Gegeben ist:$$0\le y\le 1\quad;\quad y^2\le x\le 1$$Wir können die dadurch festgelegen Punkte auch beschreiben durch:$$0\le x\le 1\quad;\quad 0\le y\le\sqrt x$$Damit kannst du die Integrationsreihenfolge nun vertauschen:$$I=\int\limits_0^1dx\int\limits_0^{\sqrt x}dy\,y\cos(x^2)=\int\limits_0^1dx\left[\frac{y^2}{2}\cos(x^2)\right]_{y=0}^{\sqrt x}=\int\limits_0^1\frac{x}{2}\cos(x^2)\,dx$$$$\phantom{I}=\frac{1}{4}\int\limits_0^12x\cos(x^2)\,dx=\frac{1}{4}\left[\sin(x^2)\right]_0^1=\frac{1}{4}\sin(1)$$

Avatar von 148 k 🚀

Danke dir aber ich verstehe immer noch nicht ganz wie man auf diese neuen Grenzen kommt.

Könntest du es mir etwas genauer erklären ?

Aloha :)

Laut Aufgabenstellung gilt: \(\quad 0\le y\le 1\quad;\quad y^2\le x\le 1\)

Das können wir in 4 Bedingungen aufteilen:

1)\(\quad 0\le y\)

2)\(\quad y\le 1\)

3)\(\quad y^2\le x\)

4)\(\quad x\le 1\)

Eine Quadratzahl ist nie negativ, daher ist \(0\le y^2\). Zusammen mit (3) haben wir also:$$0\le y^2\le x$$Damit haben wir gefunden:

a)\(\quad 0\le x\)

Die Bedingung (4) übernehmen wir als (b):

b)\(\quad x\le 1\)

und die Bedingung (1) übernehmen wir als (c):

c)\(\quad 0\le y\)

Nach (3) ist \(y^2\le x\). Das heißt \(-\sqrt x\le y\le\sqrt x\). Da nach (c) aber \(0\le y\) sein muss, haben wir bereits eine höhere Untergrenze für \(y\) als \(-\sqrt x\), nämlich die \(0\). Als Obergrenze bleibt:

d)\(\quad y\le \sqrt x\)

Fassen wir (a) bis (d) zusammen haben wir:$$0\le x\le 1\quad;\quad 0\le y\le \sqrt x$$


Ich danke dir.

Das heißt, dass ich einfach nur ai und bi finden muss mit jeweils x und y in der Mitte um so

\( a1 \leq y \leq a2 \quad ; \quad b1 \leq x \leq b2 \)

Wieso kann ich denn nicht "theoretisch"

\( 0 \leq y \leq 1 \quad ; \quad 0 \leq x \leq 1\) nehmen ?

Sollte ja auch passen ?

Visuell habe ich es jetzt verstanden.

Ein anderes Problem?

Stell deine Frage

Willkommen bei der Mathelounge! Stell deine Frage einfach und kostenlos

x
Made by a lovely community